Universal and Existential Qualifiers

Click For Summary
SUMMARY

The discussion focuses on expressing the statement "There is no largest irrational number" using quantifiers within the sets of Real and Natural Numbers. Participants propose formulations using the existential quantifier (##\exists##) and universal quantifier (##\forall##) to illustrate the existence of irrational numbers and their properties. A suggested expression is ##(\forall x \in \Re)(\exists y \in \Re) \wedge (\exists p,q,r,s \in \mathbb{N}) \ni [(\frac{p}{q} \neq x) \wedge (\frac{r}{s} \neq y) \wedge (y > x)]##, indicating that for any irrational number, a larger one can always be found. The complexity of the expression reflects the challenge of formalizing this mathematical concept.

PREREQUISITES
  • Understanding of quantifiers: universal (##\forall##) and existential (##\exists##)
  • Familiarity with the sets of Real (##\Re##) and Natural Numbers (##\mathbb{N}##)
  • Basic knowledge of irrational numbers and their properties
  • Experience with mathematical notation and expressions
NEXT STEPS
  • Research the properties of irrational numbers and their relation to rational numbers
  • Study advanced quantifier logic and its applications in mathematical proofs
  • Explore the concept of cardinality in sets, particularly between rational and irrational numbers
  • Learn about formal mathematical writing and expression of complex statements
USEFUL FOR

Mathematicians, students of mathematics, and anyone interested in formal logic and the properties of numbers, particularly those studying real analysis or set theory.

TyroneTheDino
Messages
46
Reaction score
1

Homework Statement


Express the following statement using only quantifiers. (You may only use the set of Real and Natural Numbers)

1. There is no largest irrational number.

Homework Equations


##\forall=## for all
##\exists##=there exists

The Attempt at a Solution


I express the existence of irrational numbers by saying
##(\exists x \in \Re)(\forall p,q \in\mathbb{N})(\frac{p}{q}\neq x)##

But now saying that x is not the largest irrational number is tricky to me. The book i am using said the answer would look quite complex.

To prove that there is a bigger irrational number I begin by stating that another irrational number exists, and prove that is bigger.

My thinking is that If I write:

##(\forall x \in \Re)(\exists y\in\Re)\wedge(\exists p,q,r,s \in \mathbb{N})\ni[{(\frac{p}{q}\neq x) \wedge(\frac{r}{s}\neq y)}\wedge( y>x)]##

It proves that there is always a bigger irrational number than the one that is being considered, but I'm not completely sure my reasoning makes sense
 
Physics news on Phys.org
TyroneTheDino said:

The Attempt at a Solution


I express the existence of irrational numbers by saying
##(\exists x \in \Re)(\forall p,q \in\mathbb{N})(\frac{p}{q}\neq x)##
The real number -1 is a problem. The natural number 0 is too.

TyroneTheDino said:
But now saying that x is not the largest irrational number is tricky to me. The book i am using said the answer would look quite complex.
How would you say it if you can use other sets than ##\mathbb R## and ##\mathbb N##? I would start with that, and then try to rewrite the statement using only those sets.
 
TyroneTheDino said:

Homework Statement


Express the following statement using only quantifiers. (You may only use the set of Real and Natural Numbers)

1. There is no largest irrational number.

Homework Equations


##\forall=## for all
##\exists##=there exists

The Attempt at a Solution


I express the existence of irrational numbers by saying
##(\exists x \in \Re)(\forall p,q \in\mathbb{N})(\frac{p}{q}\neq x)##

But now saying that x is not the largest irrational number is tricky to me. The book i am using said the answer would look quite complex.

To prove that there is a bigger irrational number I begin by stating that another irrational number exists, and prove that is bigger.

My thinking is that If I write:

##(\forall x \in \Re)(\exists y\in\Re)\wedge(\exists p,q,r,s \in \mathbb{N})\ni[{(\frac{p}{q}\neq x) \wedge(\frac{r}{s}\neq y)}\wedge( y>x)]##

It proves that there is always a bigger irrational number than the one that is being considered, but I'm not completely sure my reasoning makes sense

Note that you are not proving this, but only trying to express the statement. You can also express statements that are false such as "there are no irrational numbers".
 
Question: A clock's minute hand has length 4 and its hour hand has length 3. What is the distance between the tips at the moment when it is increasing most rapidly?(Putnam Exam Question) Answer: Making assumption that both the hands moves at constant angular velocities, the answer is ## \sqrt{7} .## But don't you think this assumption is somewhat doubtful and wrong?

Similar threads

  • · Replies 7 ·
Replies
7
Views
2K
Replies
20
Views
4K
  • · Replies 8 ·
Replies
8
Views
2K
  • · Replies 3 ·
Replies
3
Views
2K
  • · Replies 3 ·
Replies
3
Views
1K
  • · Replies 3 ·
Replies
3
Views
2K
  • · Replies 13 ·
Replies
13
Views
4K
  • · Replies 9 ·
Replies
9
Views
2K
Replies
1
Views
1K
Replies
3
Views
2K